The Triangle Inequality: How to Prove It?

  • Thread starter Thread starter Saladsamurai
  • Start date Start date
  • Tags Tags
    Proofs Work
Click For Summary
The discussion focuses on proving the Triangle Inequality, specifically the statements |x| - |y| ≤ |x - y| and |(|x| - |y|)| ≤ |x - y|. The first part is approached by squaring both sides and applying properties of absolute values and inequalities. The second part is derived from the first, emphasizing that the absolute value of the difference can be expressed in terms of the original inequality. Participants clarify the conditions under which the equality holds and reference the Cauchy-Schwarz inequality in their reasoning. The conversation highlights the logical steps necessary to complete the proof effectively.
Saladsamurai
Messages
3,009
Reaction score
7

Homework Statement



Prove the following:

(i) ##|x|-|y| \le |x-y|##

and

(ii) ##|(|x|-|y|)| \le |x-y|\qquad## (Why does this immediately follow from (i) ?)

Homework Equations



##|z| = \sqrt{z^2}##

The Attempt at a Solution



(i) ##(|x|-|y|)^2 = |x|^2 - 2|x||y| + |y|^2 = x^2 - 2|x||y| + y^2 \le x^2 - 2xy + y^2= (x-y)^2 \implies \boxed{|x|-|y| \le |x-y|.}##(ii) For this part, I looked at the question "Why does this immediately follow from (i)" for inspiration and saw that if I could show that ##|(|x|-|y|)| \le |x-y|## then the proof is complete by transitivity.

Is it as simple as:

##|(|x|-|y|)| = \sqrt{(|(|x|-|y|)|)^2} = \sqrt{(|x|-|y|)^2} = |x|-|y|?##

I think that it is, but it is getting late :redface:
 
Physics news on Phys.org
If |x|> |y|, (ii) is identical to (i). If |y|< |x|, swap the two.
 
Saladsamurai said:
Is it as simple as:

##|(|x|-|y|)| = \sqrt{(|(|x|-|y|)|)^2} = \sqrt{(|x|-|y|)^2} = |x|-|y|?##

No, it isn't. ##\sqrt{(|x|-|y|)^2} = |x|-|y|## only when ##|x|\ge |y|##.
 
Saladsamurai said:
(i) ##(|x|-|y|)^2 = |x|^2 - 2|x||y| + |y|^2 = x^2 - 2|x||y| + y^2 \le x^2 - 2xy + y^2= (x-y)^2 \implies \boxed{|x|-|y| \le |x-y|.}##

Remember that ##\sqrt{a^2}=|a|## and if ##0\le a\le b## then ##\sqrt a\le \sqrt b##. So how can you change the above conclusion?
 
Isn't this what is called the kaushy swartz?
 
Question: A clock's minute hand has length 4 and its hour hand has length 3. What is the distance between the tips at the moment when it is increasing most rapidly?(Putnam Exam Question) Answer: Making assumption that both the hands moves at constant angular velocities, the answer is ## \sqrt{7} .## But don't you think this assumption is somewhat doubtful and wrong?

Similar threads

Replies
2
Views
1K
  • · Replies 3 ·
Replies
3
Views
907
  • · Replies 14 ·
Replies
14
Views
2K
  • · Replies 6 ·
Replies
6
Views
2K
Replies
8
Views
2K
Replies
4
Views
2K
Replies
3
Views
2K
Replies
1
Views
1K
  • · Replies 2 ·
Replies
2
Views
2K